LSAT and Law School Admissions Forum

Get expert LSAT preparation and law school admissions advice from PowerScore Test Preparation.

User avatar
 mainebaltazar
  • Posts: 3
  • Joined: Feb 09, 2022
|
#94061
Hi,

Please help me understand why my thought process was wrong on this.

The stimulus says concludes that "antitheft devices do not protect cars against thieves" because of statistics that show that "cars with alarms or other antitheft devices are more likely to be stolen or broken into." I determined that the flaw in this reasoning could be that they failed to recognize that the characteristic in question (having an alarm or antitheft device) is likely to come with something else that actually leads to the conclusion. For example, carjackings and robberies involving cars that are equipped with alarms and antitheft devices are probably more likely to be reported because they have devices that signal that an incident occurred.

So my idea was to look for an answer where the reasoning is wrong because there was a failure to consider that the characteristic in question might be associated with something else that could lead to the conclusion. I chose (D) "Since youngsters who read voraciously are more likely to have defective vision than youngsters who do not read very much, it follows that children who do not like to read usually have perfect vision." because I thought that young people who read voraciously could be more prone to poor vision because a lot of reading can be straining on the eyes, and such straining of the eyes is what can cause poor vision, not the fact that they are avid readers.

So in both the stimulus and choice (D), something else could've contributed to the effect and therefore we should not come to a conclusion about the first characteristic mentioned (having an antitheft device (stim) or reading a lot (stim)).
 Adam Tyson
PowerScore Staff
  • PowerScore Staff
  • Posts: 5271
  • Joined: Apr 14, 2011
|
#94094
While your initial approach of focusing on the possibility that some other causal factor may have been overlooked was good, mainebaltazar, answer D doesn't follow through on that because the author doesn't draw a conclusion that the cause being analyzed is not having any effect.

When choosing a Parallel Reasoning or Parallel Flaw answer, you should make sure that the answer choice has a conclusion that is very similar in its strength and logical form to the conclusion in the argument. In this case, that would mean the correct answer has to be that one thing is not causing a second thing, just as happened in the stimulus.

The correct answer also needs to match the degree of certainty that was in the stimulus. Answer D used the word "usually" in its conclusion, which is not as certain as the conclusion in the stimulus. Our author said nothing in the conclusion about what is usually true, but instead concluded with 100% certainty that the devices do not protect against theft.

Matching the flaw is more than just matching some element of the flaw. It also must use the same kind of reasoning and the same strength of language. Answer C has it all: 1) there could be some other causal factor involved that contributes to these people buying more books (perhaps the cause is a love of reading, and that makes them use the library AND buy more books); 2) the type of conclusion is the same (X does not cause Y); and 3) the strength is the same (100% certain that X is not having that effect; no use of softer language of probability like "usually" or "probably" or "tends to.")

For more on our approach, check out Episode 4 of the Powerscore LSAT Podcast, which you can find wherever you get podcasts. There's also a link to the recording accessible through the Powerscore LSAT Blog here:

https://blog.powerscore.com/lsat/the-po ... questions/

Get the most out of your LSAT Prep Plus subscription.

Analyze and track your performance with our Testing and Analytics Package.